Você está na página 1de 15

MAE0001 - Lista de Exercı́cios 2

Prof. Bruno Castro

27 de janeiro de 2021

Livro Probabilidade e Veriáveis Aleatórias (Ross) - Exercı́cios Teóricos 3.4, 3.5, 3.12, 3.16,
3.20, 3.22, 3.28 (página 111 à 113) Exercı́cios “ Self-Test”3.4, 3.6, 3.21, 3.22, (página 114 e 115).

Livro A Course in Mathematical Statistics (Roussas) - Exercı́cios 2.2.1, 2.2.2, 2.2.7, 2.2.10,
2.2.14, 2.2.18 (páginas 25 à 27), 2.4.35 e 2.4.36 (página 45).

Livro Probabilidade: um curso em nı́vel intermediário (James) - Exercı́cios 14, 16, 17, 19,
21, 22 (páginas 30 à 32)

1. Sejam A e B conjuntos de números naturais com |A| = p e |B| = n.

(a) Quantas são as funções f : A → B?


(b) Quantas são as funções f : A → B injetoras?
(c) Quantas são as funções f : A → B sobrejetoras?
(d) Quantas são as funções f : A → B não-decrescentes?

2. Um armário contém n pares diferentes de sapatos. Se 2r sapatos são escolhidos ao acaso (com
2r < n). De quantas maneiras podemos organizar os sapatos de forma que:

(a) não exista par algum completo


(b) exista exatamente um par completo.
(c) existam exatamente dois pares completos

3. Se n homens entre os quais A e B estão numa fila, de quantos modos podemos organizar
os homens em uma fila de forma que existem exatamente r homens entre A e B? Se eles
permanecerem em pé num cı́rculo ao invés de uma fila?

4. De quantas maneiras é possı́vel colocar 6 anéis diferentes em 4 dedos?

5. Uma pessoa possui 5 livros diferentes de Matemática, 2 livros diferentes de Quı́micae 3 li-
vros diferentes de Fı́sica, que serão dispostos aleatoriamente em uma prateleira. Calcule as
probabilidades de que

a) os livros de cada assunto fiquem juntos.

1
b) os livros de Matemática não fiquem todos juntos.
c) os livros de Fı́sica fiquem todos separados.
d) os livros de um mesmo assunto apareçam em ordem alfabética, mas não necessariamente
adjacentes.

6. Distribuı́mos 12 bolas em 5 caixas numeradas 1, 2, 3, 4, 5. Calcule a probabilidade da caixa


1 conter exatamente 3 bolas se

a) as bolas são distinguı́veis.


b) as bolas são indistinguı́veis.

7. A caixa I contém 4 bolas e 2 pretas, a caixa II contém 3 bolas brancas e 1 preta e a caixa III
contém 1 bola branca e 2 pretas.

a) Extrai-se uma bola da caixa. Determine a probabilidade de que todas as bolas sejam
brancas.
b) Seleciona-se uma caixa e dela extra-se uma bola. Determine a probabilidade de que a
bola extraı́da seja branca.
c) Calcule a probabilidade de que a primeira caixa tenha sido escolhida, dado que a bola
extraı́da é branca.

2
Theoretical Exercises 111

To do so, multiply the sums and show that, for all 3.8. Let A, B, and C be events relating to the experi-
pairs i, j, the coefficient of the term ni nj is greater ment of rolling a pair of dice.
in the expression on the left than in the one on the (a) If
right.
3.4. A ball is in any one of n boxes and is in the ith box P(A|C) > P(B|C) and P(A|Cc ) > P(B|Cc )
with probability Pi . If the ball is in box i, a search of
either prove that P(A) > P(B) or give a coun-
that box will uncover it with probability αi . Show
terexample by defining events A, B, and C for
that the conditional probability that the ball is in
which that relationship is not true.
box j, given that a search of box i did not uncover
(b) If
it, is
Pj
if j Z i P(A|C) > P(A|Cc ) and P(B|C) > P(B|Cc )
1 − αi Pi
(1 − αi )Pi either prove that P(AB|C) > P(AB|Cc ) or
if j = i give a counterexample by defining events
1 − αi Pi
A, B, and C for which that relationship is not
3.5. An event F is said to carry negative information true.
about an event E, and we write F R E, if Hint: Let C be the event that the sum of a pair of
dice is 10; let A be the event that the first die lands
P(E|F) … P(E) on 6; let B be the event that the second die lands
on 6.
Prove or give counterexamples to the following 3.9. Consider two independent tosses of a fair coin. Let
assertions: A be the event that the first toss results in heads, let
(a) If F R E, then E R F. B be the event that the second toss results in heads,
(b) If F R E and E R G, then F R G. and let C be the event that in both tosses the coin
(c) If F R E and G R E, then FG R E. lands on the same side. Show that the events A, B,
Repeat parts (a), (b), and (c) when R is replaced and C are pairwise independent—that is, A and B
by Q, where we say that F carries positive informa- are independent, A and C are independent, and B
tion about E, written F Q E, when P(E|F) Ú P(E). and C are independent—but not independent.
3.6. Prove that if E1 , E2 , . . . , En are independent 3.10. Consider a collection of n individuals. Assume that
events, then each person’s birthday is equally likely to be any of
the 365 days of the year and also that the birthdays

n
are independent. Let Ai,j , i Z j, denote the event
P(E1 ∪ E2 ∪ · · · ∪ En ) = 1 − [1 − P(Ei )] that persons i and j have the same birthday. Show
i=1 that these events are pairwise independent, but not
independent. That is,show  that Ai,j and Ar,s are
3.7. (a) An urn contains n white and m black balls.
n
The balls are withdrawn one at a time until independent, but the events Ai,j , i Z j are not
only those of the same color are left. Show 2
that, with probability n/(n + m), they are all independent.
white. 3.11. In each of n independent tosses of a coin, the coin
Hint: Imagine that the experiment continues lands on heads with probability p. How large need
until all the balls are removed, and consider n be so that the probability of obtaining at least
the last ball withdrawn. one head is at least 12 ?
(b) A pond contains 3 distinct species of fish, 3.12. Show that 0 … ai … 1, i = 1, 2, . . ., then
which we will call the Red, Blue, and Green ⎡ ⎤
fish. There are r Red, b Blue, and g Green fish. q 
i−1 q
⎢ ⎥
Suppose that the fish are removed from the ⎣ i
a (1 − a )
j ⎦ + (1 − ai ) = 1
pond in a random order. (That is, each selec- i=1 j=1 i=1
tion is equally likely to be any of the remain-
ing fish.) What is the probability that the Red Hint: Suppose that an infinite number of coins are
fish are the first species to become extinct in to be flipped. Let ai be the probability that the ith
the pond? coin lands on heads, and consider when the first
Hint: Write P{R} = P{RBG} + P{RGB}, head occurs.
and compute the probabilities on the right 3.13. The probability of getting a head on a single toss
by first conditioning on the last species to be of a coin is p. Suppose that A starts and continues
removed. to flip the coin until a tail shows up, at which point
112 Chapter 3 Conditional Probability and Independence

B starts flipping. Then B continues to flip until a 3.18. Let Qn denote the probability that no run of 3 con-
tail comes up, at which point A takes over, and so secutive heads appears in n tosses of a fair coin.
on. Let Pn,m denote the probability that A accu- Show that
mulates a total of n heads before B accumulates
1 1 1
m. Show that Qn = Qn−1 + Qn−2 + Qn−3
2 4 8
Pn,m = pPn−1,m + (1 − p)(1 − Pm,n ) Q0 = Q 1 = Q 2 = 1

∗ 3.14. Suppose that you are gambling against an infinitely Find Q8 .


rich adversary and at each stage you either win Hint: Condition on the first tail.
or lose 1 unit with respective probabilities p and 3.19. Consider the gambler’s ruin problem, with the
1 − p. Show that the probability that you eventu- exception that A and B agree to play no more than
ally go broke is n games. Let Pn,i denote the probability that A
winds up with all the money when A starts with
1 if p … 1 i and B starts with N − i. Derive an equation
2
for Pn,i in terms of Pn−1, i+1 and Pn−1, i−1 , and
(q/p)i if p > 1
2 compute P7, 3 , N = 5.
3.20. Consider two urns, each containing both white
where q = 1 − p and where i is your initial fortune.
and black balls. The probabilities of drawing white
3.15. Independent trials that result in a success with balls from the first and second urns are, respec-
probability p are successively performed until a tively, p and p . Balls are sequentially selected with
total of r successes is obtained. Show that the prob- replacement as follows: With probability α, a ball
ability that exactly n trials are required is is initially chosen from the first urn, and with prob-
  ability 1 − α, it is chosen from the second urn. The
n − 1
pr (1 − p)n−r subsequent selections are then made according to
r − 1
the rule that whenever a white ball is drawn (and
replaced), the next ball is drawn from the same
Use this result to solve the problem of the points
urn, but when a black ball is drawn, the next ball is
(Example 4j).
taken from the other urn. Let αn denote the prob-
Hint: In order for it to take n trials to obtain r suc-
ability that the nth ball is chosen from the first urn.
cesses, how many successes must occur in the first
Show that
n − 1 trials?
3.16. Independent trials that result in a success with αn+1 = αn (p + p − 1) + 1 − p n Ú 1
probability p and a failure with probability 1 −
p are called Bernoulli trials. Let Pn denote the and use this formula to prove that
probability that n Bernoulli trials result in an even  
number of successes (0 being considered an even 1 − p 1 − p
αn = + α −
number). Show that 2 − p − p 2 − p − p
Pn = p(1 − Pn−1 ) + (1 − p)Pn−1 n Ú 1 * (p + p − 1)n−1

and use this formula to prove (by induction) that Let Pn denote the probability that the nth
ball selected is white. Find Pn . Also, compute
1 + (1 − 2p)n limn→q αn and limn→q Pn .
Pn =
2 3.21. The Ballot Problem. In an election, candidate A
receives n votes and candidate B receives m votes,
3.17. Suppose that n independent trials are performed, where n > m. Assuming that all of the (n +
with trial i being a success with probability 1/(2i + m)!/n! m! orderings of the votes are equally likely,
1). Let Pn denote the probability that the total let Pn,m denote the probability that A is always
number of successes that result is an odd number. ahead in the counting of the votes.
(a) Find Pn for n = 1, 2, 3, 4, 5. (a) Compute P2,1 , P3,1 , P3,2 , P4,1 , P4,2 , P4,3 .
(b) Conjecture a general formula for Pn . (b) Find Pn,1 , Pn,2 .
(c) Derive a formula for Pn in terms of Pn−1 . (c) On the basis of your results in parts (a) and
(d) Verify that your conjecture in part (b) satisfies (b), conjecture the value of Pn,m .
the recursive formula in part (d). Because the (d) Derive a recursion for Pn,m in terms of Pn−1,m
recursive formula has a unique solution, this and Pn,m−1 by conditioning on who receives
then proves that your conjecture is correct. the last vote.
Theoretical Exercises 113
 
(e) Use part (d) to verify your conjecture in part n
(c) by an induction proof on n + m. to be won by either contestant. Number the
k
3.22. As a simplified model for weather forecasting, sup- sets of k contestants, and let Bi denote the event
pose that the weather (either wet or dry) tomor- that no contestant beat all of the k players in
row will be the same as the weather today with the
 ith set.
 Then use Boole’s inequality to bound
probability p. Show that the weather is dry on Jan-

uary 1, then Pn , the probability that it will be dry n P Bi .


i
days later, satisfies
3.25. Prove directly that
Pn = (2p − 1)Pn−1 + (1 − p) n Ú 1
P(E|F) = P(E|FG)P(G|F) + P(E|FGc )P(Gc |F)
P0 = 1
3.26. Prove the equivalence of Equations (5.11) and
Prove that (5.12).
1 1 3.27. Extend the definition of conditional independence
Pn = + (2p − 1)n n Ú 0 to more than 2 events.
2 2
3.28. Prove or give a counterexample. If E1 and E2 are
3.23. A bag contains a white and b black balls. Balls independent, then they are conditionally indepen-
are chosen from the bag according to the following dent given F.
method: 3.29. In Laplace’s rule of succession (Example 5e),
1. A ball is chosen at random and is discarded. show that if the first n flips all result in heads,
then the conditional probability that the next m
2. A second ball is then chosen. If its color is
flips also result in all heads is (n + 1)/(n +
different from that of the preceding ball, it is
m + 1).
replaced in the bag and the process is repeated
3.30. In Laplace’s rule of succession (Example 5e), sup-
from the beginning. If its color is the same, it is
pose that the first n flips resulted in r heads and
discarded and we start from step 2.
n − r tails. Show that the probability that the
In other words, balls are sampled and discarded (n + 1)st flip turns up heads is (r + 1)/(n + 2). To
until a change in color occurs, at which point the do so, you will have to prove and use the identity
last ball is returned to the urn and the process * 1
starts anew. Let Pa,b denote the probability that n!m!
yn (1 − y)m dy =
the last ball in the bag is white. Prove that 0 (n + m + 1)!
1 Hint: To prove the identity, let C(n, m) =
Pa,b = -1 n
0 y (1 − y) dy. Integrating by parts yields
2 m

Hint: Use induction on k K a + b. m


∗ 3.24. C(n, m) = C(n + 1, m − 1)
 
A round-robin tournament of n contestants is a n + 1
n
tournament in which each of the pairs of
2 Starting with C(n, 0) = 1/(n + 1), prove the iden-
contestants play each other exactly once, with the tity by induction on m.
outcome of any play being that one of the contes- 3.31. Suppose that a nonmathematical, but philosophi-
tants wins and the other loses. For a fixed integer cally minded, friend of yours claims that Laplace’s
k, k < n, a question of interest is whether it is pos- rule of succession must be incorrect because it can
sible that the tournament outcome is such that, for lead to ridiculous conclusions. “For instance,” says
every set of k players, there is a player who beat he, “the rule states that if a boy is 10 years old,
each member of that set. Show that if having lived 10 years, the boy has probability 11
12 of
   k n−k living another year. On the other hand, if the boy
n 1 has an 80-year-old grandfather, then, by Laplace’s
1 − < 1
k 2 rule, the grandfather has probability 81 82 of sur-
viving another year. However, this is ridiculous.
then such an outcome is possible. Clearly, the boy is more likely to survive an addi-
Hint: Suppose that the results of the games are tional year than the grandfather is.” How would
independent and that each game is equally likely you answer your friend?
114 Chapter 3 Conditional Probability and Independence

SELF-TEST PROBLEMS AND EXERCISES

3.1. In a game of bridge, West has no aces. What is the 3.8. Show that
probability of his partner’s having (a) no aces? (b)
P(H|E) P(H) P(E|H)
2 or more aces? (c) What would the probabilities =
be if West had exactly 1 ace? P(G|E) P(G) P(E|G)
3.2. The probability that a new car battery functions Suppose that, before new evidence is observed, the
for over 10,000 miles is .8, the probability that it hypothesis H is three times as likely to be true as
functions for over 20,000 miles is .4, and the prob- is the hypothesis G. If the new evidence is twice
ability that it functions for over 30,000 miles is .1. If as likely when G is true than it is when H is true,
a new car battery is still working after 10,000 miles, which hypothesis is more likely after the evidence
what is the probability that has been observed?
(a) its total life will exceed 20,000 miles? 3.9. You ask your neighbor to water a sickly plant
(b) its additional life will exceed 20,000 miles? while you are on vacation. Without water, it will
3.3. How can 20 balls, 10 white and 10 black, be put die with probability .8; with water, it will die with
into two urns so as to maximize the probability of probability .15. You are 90 percent certain that
drawing a white ball if an urn is selected at random your neighbor will remember to water the plant.
and a ball is drawn at random from it? (a) What is the probability that the plant will be
3.4. Urn A contains 2 white balls and 1 black ball, alive when you return?
whereas urn B contains 1 white ball and 5 black (b) If the plant is dead upon your return, what is
balls. A ball is drawn at random from urn A and the probability that your neighbor forgot to
placed in urn B. A ball is then drawn from urn B. water it?
It happens to be white. What is the probability that 3.10. Six balls are to be randomly chosen from an urn
the ball transferred was white? containing 8 red, 10 green, and 12 blue balls.
(a) What is the probability at least one red ball is
3.5. An urn has r red and w white balls that are ran-
chosen?
domly removed one at a time. Let Ri be the event
(b) Given that no red balls are chosen, what is the
that the ith ball removed is red. Find
conditional probability that there are exactly
(a) P(Ri )
2 green balls among the 6 chosen?
(b) P(R5 |R3 )
3.11. A type C battery is in working condition with prob-
(c) P(R3 |R5 )
ability .7, whereas a type D battery is in work-
3.6. An urn contains b black balls and r red balls. One ing condition with probability .4. A battery is ran-
of the balls is drawn at random, but when it is domly chosen from a bin consisting of 8 type C and
put back in the urn, c additional balls of the same 6 type D batteries.
color are put in with it. Now, suppose that we (a) What is the probability that the battery
draw another ball. Show that the probability that works?
the first ball was black, given that the second ball (b) Given that the battery does not work, what is
drawn was red, is b/(b + r + c). the conditional probability that it was a type
3.7. A friend randomly chooses two cards, without C battery?
replacement, from an ordinary deck of 52 playing 3.12. Maria will take two books with her on a trip. Sup-
cards. In each of the following situations, deter- pose that the probability that she will like book 1
mine the conditional probability that both cards is .6, the probability that she will like book 2 is .5,
are aces. and the probability that she will like both books
(a) You ask your friend if one of the cards is the is .4. Find the conditional probability that she will
ace of spades, and your friend answers in the like book 2 given that she did not like book 1.
affirmative. 3.13. Balls are randomly removed from an urn that ini-
(b) You ask your friend if the first card selected tially contains 20 red and 10 blue balls.
is an ace, and your friend answers in the affir- (a) What is the probability that all of the red balls
mative. are removed before all of the blue ones have
(c) You ask your friend if the second card been removed?
selected is an ace, and your friend answers in Now suppose that the urn initially contains 20
the affirmative. red, 10 blue, and 8 green balls.
(d) You ask your friend if either of the cards (b) Now what is the probability that all of the red
selected is an ace, and your friend answers in balls are removed before all of the blue ones
the affirmative. have been removed?
Self-Test Problems and Exercises 115

(c) What is the probability that the colors are man out. If there is no odd man out, the players flip
depleted in the order blue, red, green? again and continue to do so until they get an odd
(d) What is the probability that the group of blue man out. What is the probability that A will be the
balls is the first of the three groups to be odd man?
removed? 3.20. Suppose that there are n possible outcomes of
3.14. A coin having probability .8 of landing on heads a trial, with outcome i resulting with probability
is flipped. A observes the result—either heads or
n
pi , i = 1, . . . , n, pi = 1. If two independent tri-
tails—and rushes off to tell B. However, with prob- i=1
ability .4, A will have forgotten the result by the als are observed, what is the probability that the
time he reaches B. If A has forgotten, then, rather result of the second trial is larger than that of the
than admitting this to B, he is equally likely to tell first?
B that the coin landed on heads or that it landed 3.21. If A flips n + 1 and B flips n fair coins, show that
tails. (If he does remember, then he tells B the cor- the probability that A gets more heads than B is 12 .
rect result.) Hint: Condition on which player has more heads
(a) What is the probability that B is told that the after each has flipped n coins. (There are three
coin landed on heads? possibilities.)
(b) What is the probability that B is told the cor- 3.22. Prove or give counterexamples to the following
rect result? statements:
(c) Given that B is told that the coin landed on (a) If E is independent of F and E is independent
heads, what is the probability that it did in fact of G, then E is independent of F ∪ G.
land on heads? (b) If E is independent of F, and E is independent
3.15. In a certain species of rats, black dominates over of G, and FG = Ø, then E is independent of
brown. Suppose that a black rat with two black F ∪ G.
parents has a brown sibling. (c) If E is independent of F, and F is independent
(a) What is the probability that this rat is a pure of G, and E is independent of FG, then G is
black rat (as opposed to being a hybrid with independent of EF.
one black and one brown gene)? 3.23. Let A and B be events having positive probabil-
(b) Suppose that when the black rat is mated with ity. State whether each of the following statements
a brown rat, all 5 of their offspring are black. is (i) necessarily true, (ii) necessarily false, or (iii)
Now what is the probability that the rat is a possibly true.
pure black rat? (a) If A and B are mutually exclusive, then they
3.16. (a) In Problem 3.65b, find the probability that a are independent.
current flows from A to B, by conditioning on (b) If A and B are independent, then they are
whether relay 1 closes. mutually exclusive.
(b) Find the conditional probability that relay 3 is (c) P(A) = P(B) = .6, and A and B are mutually
closed given that a current flows from A to B. exclusive.
3.17. For the k-out-of-n system described in (d) P(A) = P(B) = .6, and A and B are indepen-
Problem 3.67, assume that each component dent.
independently works with probability 12 . Find the 3.24. Rank the following from most likely to least likely
conditional probability that component 1 is work- to occur:
ing, given that the system works, when 1. A fair coin lands on heads.
(a) k = 1, n = 2;
2. Three independent trials, each of which is a suc-
(b) k = 2, n = 3.
cess with probability .8, all result in successes.
3.18. Mr. Jones has devised a gambling system for win-
ning at roulette. When he bets, he bets on red and 3. Seven independent trials, each of which is a suc-
places a bet only when the 10 previous spins of cess with probability .9, all result in successes.
the roulette have landed on a black number. He 3.25. Two local factories, A and B, produce radios. Each
reasons that his chance of winning is quite large radio produced at factory A is defective with prob-
because the probability of 11 consecutive spins ability .05, whereas each one produced at factory B
resulting in black is quite small. What do you think is defective with probability .01. Suppose you pur-
of this system? chase two radios that were produced at the same
3.19. Three players simultaneously toss coins. The coin factory, which is equally likely to have been either
tossed by A(B)[C] turns up heads with probability factory A or factory B. If the first radio that you
P1 (P2 )[P3 ]. If one person gets an outcome differ- check is defective, what is the conditional proba-
ent from those of the other two, then he is the odd bility that the other one is also defective?
2.4 Combinatorial
Exercises
Results 25

is a partition of S. In fact,

(A ∩ B ) ∩ (A
i j i′ )
∩ B j ′ = ∅ if (i, j ) ≠ (i′, j ′)
and

∑ ( A ∩ B ) = ∑ ∑ ( A ∩ B ) = ∑ A = S.
i j i j i
i, j i j i

The expression P(Ai ∩ Bj) is called the joint probability of Ai and Bj. On the
other hand, from
Ai = ∑ Ai ∩ Bj
j
( ) and Bj = ∑ Ai ∩ Bj ,
i
( )
we get

( ) (
P Ai = ∑ P Ai ∩ B j = ∑ P Ai B j P B j ,
j
) j
( )( )
provided P(Bj) > 0, j = 1, 2, . . . , and

( ) (
P B j = ∑ P Ai ∩ B j = ∑ P B j Ai P Ai ,
i
) i
( )( )
provided P(Ai) > 0, i = 1, 2, . . . . The probabilities P(Ai), P(Bj) are called
marginal probabilities. We have analogous expressions for the case of more
than two partitions of S.

Exercises
2.2.1 If P(A|B) > P(A), then show that P(B|A) > P(B) (P(A)P(B) > 0).
2.2.2 Show that:
i) P(Ac|B) = 1 − P(A|B);
ii) P(A ∪ B|C) = P(A|C) + P(B|C) − P(A ∩ B|C).
Also show, by means of counterexamples, that the following equations need
not be true:
iii) P(A|Bc) = 1 − P(A|B);
iv) P(C|A + B) = P(C|A) + P(C|B).
2.2.3 If A ∩ B = ∅ and P(A + B) > 0, express the probabilities P(A|A + B)
and P(B|A + B) in terms of P(A) and P(B).
2.2.4 Use induction to prove Theorem 3.
2.2.5 Suppose that a multiple choice test lists n alternative answers of which
only one is correct. Let p, A and B be defined as in Example 2 and find Pn(A|B)
26 2 Some Probabilistic Concepts and Results

in terms of n and p. Next show that if p is fixed but different from 0 and 1, then
Pn(A|B) increases as n increases. Does this result seem reasonable?
2.2.6 If Aj, j = 1, 2, 3 are any events in S, show that {A1, Ac1 ∩ A2, Ac1 ∩ Ac2 ∩
A3, (A1 ∪ A2 ∪ A3)c} is a partition of S.
2.2.7 Let {Aj, j = 1, . . . , 5} be a partition of S and suppose that P(Aj) = j/15
and P(A|Aj) = (5 − j)/15, j = 1, . . . , 5. Compute the probabilities P(Aj|A),
j = 1, . . . , 5.
2.2.8 A girl’s club has on its membership rolls the names of 50 girls with the
following descriptions:
20 blondes, 15 with blue eyes and 5 with brown eyes;
25 brunettes, 5 with blue eyes and 20 with brown eyes;
5 redheads, 1 with blue eyes and 4 with green eyes.
If one arranges a blind date with a club member, what is the probability that:
i) The girl is blonde?
ii) The girl is blonde, if it was only revealed that she has blue eyes?
2.2.9 Suppose that the probability that both of a pair of twins are boys is 0.30
and that the probability that they are both girls is 0.26. Given that the probabil-
ity of a child being a boy is 0.52, what is the probability that:
i) The second twin is a boy, given that the first is a boy?
ii) The second twin is a girl, given that the first is a girl?
2.2.10 Three machines I, II and III manufacture 30%, 30% and 40%, respec-
tively, of the total output of certain items. Of them, 4%, 3% and 2%, respec-
tively, are defective. One item is drawn at random, tested and found to be
defective. What is the probability that the item was manufactured by each one
of the machines I, II and III?
2.2.11 A shipment of 20 TV tubes contains 16 good tubes and 4 defective
tubes. Three tubes are chosen at random and tested successively. What is the
probability that:
i) The third tube is good, if the first two were found to be good?
ii) The third tube is defective, if one of the other two was found to be good
and the other one was found to be defective?
2.2.12 Suppose that a test for diagnosing a certain heart disease is 95%
accurate when applied to both those who have the disease and those who do
not. If it is known that 5 of 1,000 in a certain population have the disease in
question, compute the probability that a patient actually has the disease if the
test indicates that he does. (Interpret the answer by intuitive reasoning.)
2.2.13 Consider two urns Uj, j = 1, 2, such that urn Uj contains mj white balls
and nj black balls. A ball is drawn at random from each one of the two urns and
2.4 Combinatorial
2.3 Independence
Results 27

is placed into a third urn. Then a ball is drawn at random from the third urn.
Compute the probability that the ball is black.
2.2.14 Consider the urns of Exercise 2.2.13. A balanced die is rolled and if
an even number appears, a ball, chosen at random from U1, is transferred to
urn U2. If an odd number appears, a ball, chosen at random from urn U2, is
transferred to urn U1. What is the probability that, after the above experiment
is performed twice, the number of white balls in the urn U2 remains the same?
2.2.15 Consider three urns Uj, j = 1, 2, 3 such that urn Uj contains mj white
balls and nj black balls. A ball, chosen at random, is transferred from urn U1 to
urn U2 (color unnoticed), and then a ball, chosen at random, is transferred
from urn U2 to urn U3 (color unnoticed). Finally, a ball is drawn at random
from urn U3. What is the probability that the ball is white?
2.2.16 Consider the urns of Exercise 2.2.15. One urn is chosen at random
and one ball is drawn from it also at random. If the ball drawn was white, what
is the probability that the urn chosen was urn U1 or U2?
2.2.17 Consider six urns Uj, j = 1, . . . , 6 such that urn Uj contains mj (≥ 2)
white balls and nj (≥ 2) black balls. A balanced die is tossed once and if the
number j appears on the die, two balls are selected at random from urn Uj.
Compute the probability that one ball is white and one ball is black.
2.2.18 Consider k urns Uj, j = 1, . . . , k each of which contain m white balls
and n black balls. A ball is drawn at random from urn U1 and is placed in urn
U2. Then a ball is drawn at random from urn U2 and is placed in urn U3 etc.
Finally, a ball is chosen at random from urn Uk−1 and is placed in urn Uk. A ball
is then drawn at random from urn Uk. Compute the probability that this last
ball is black.

2.3 Independence
For any events A, B with P(A) > 0, we defined P(B|A) = P(A ∩ B)/P(A). Now
P(B|A) may be >P(B), < P(B), or = P(B). As an illustration, consider an urn
containing 10 balls, seven of which are red, the remaining three being black.
Except for color, the balls are identical. Suppose that two balls are drawn
successively and without replacement. Then (assuming throughout the uni-
form probability function) the conditional probability that the second ball is
red, given that the first ball was red, is 69 , whereas the conditional probability
that the second ball is red, given that the first was black, is 79 . Without any
knowledge regarding the first ball, the probability that the second ball is red is
7
10
. On the other hand, if the balls are drawn with replacement, the probability
that the second ball is red, given that the first ball was red, is 107 . This probabil-
ity is the same even if the first ball was black. In other words, knowledge of the
event which occurred in the first drawing provides no additional information in
2.5 Product
2.4 Combinatorial
Probability Results
Spaces 45

{
A7 = s ∈ S ; s consists of 3 aces, 2 kings and exactly 7 red cards , }
A8 = {s ∈ S ; s consists of cards of all different denominations}.

2.4.35 Refer to Exercise 2.4.34 and for j = 0, 1, . . . , 4, define the events Aj


and also A as follows:

{ }
A j = s ∈ S ; s contains exactly j tens ,
A = {s ∈ S ; s contains exactly 7 red cards}.

For j = 0, 1, . . . , 4, compute the probabilities P(Aj), P(Aj|A) and also P(A);


compare the numbers P(Aj), P(Aj|A).
2.4.36 Let S be the set of all n3 3-letter words of a language and let P be the
equally likely probability function on the events of S. Define the events A, B
and C as follows:

{
A = s ∈S ; s begins with a specific letter , }
{ (
B = s ∈S ; s has the specified letter mentioned in the definition of A )
in the middle entry , }
{
C = s ∈S ; s has exactly two of its letters the same . }
Then show that:
i) P(A ∩ B) = P(A)P(B);
ii) P(A ∩ C) = P(A)P(C);
iii) P(B ∩ C) = P(B)P(C);
iv) P(A ∩ B ∩ C) ≠ P(A)P(B)P(C).
Thus the events A, B, C are pairwise independent but not mutually
independent.

2.5* Product Probability Spaces


The concepts discussed in Section 2.3 can be stated precisely by utilizing more
technical language. Thus, if we consider the experiments E1 and E2 with re-
spective probability spaces (S1, A1, P1) and (S2, A2, P2), then the compound
experiment (E1, E2) = E1 × E2 has sample space S = S1 × S2 as defined earlier.
The appropriate σ-field A of events in S is defined as follows: First define the
class C by:

{
C = A1 × A2 ; A1 ∈ A1 , A2 ∈ A2 , }
where A1 × A2 = {(s , s ); s ∈ A , s
1 2 1 1 2 }
∈ A2 .

Você também pode gostar